文章目錄
  1. 1. 非退化矩阵的判别准则
  2. 2. 克拉默公式
  3. 3. 加边子式法
  4. 4. 习题

非退化矩阵的判别准则

根据第$2$章$\S 3$定理$5$,矩阵$A\in M_n (\mathbb{R} )$的非退化性(即$\text{rank} A =n $)等价于它的可逆性.将$\S 2$定理$3$应用于关系式$AA^{-1} =A^{-1} A=E$,得到

$$\det A\cdot \det (A^{-1}) =1.$$

因而,非退化矩阵的行列式不等于零,且

$$\det (A^{-1}) =(\det A)^{-1} .$$

给出矩阵$A$,我们可以同时考察它的伴随矩阵

$$A^{\vee } =\begin{pmatrix} A_{11} & \cdots & A_{n1} \\ \cdots & \cdots & \cdots \\ A_{1n} & \cdots & A_{nn} \end{pmatrix} .$$

为了得到$A^{\vee }$,我们需要在矩阵$A$的每一个元素$a_{ij}$的位置放上它的代数余子式$A_{ij} (i,j=1,\cdots ,n )$,然后取转置.

定理1$\quad $矩阵$A\in M_n (\mathbb{R} )$是非退化的(可逆的),当且仅当$\det A\neq 0$.若$\det A\neq 0$,则

$$A^{-1} =(\det A)^{-1} A^{\vee } ,$$

或更详细地写成

$$\begin{pmatrix} a_{11} & \cdots & a_{1n} \\ \cdots & \cdots & \cdots \\ a_{n1} & \cdots & a_{nn} \end{pmatrix} ^{-1} =\begin{pmatrix} \dfrac{A_{11}}{\det A} & \cdots & \dfrac{A_{n1}}{\det A} \\ \cdots & \cdots & \cdots \\ \dfrac{A_{n1}}{\det A} & \cdots & \dfrac{A_{nn}}{\det A} \end{pmatrix}.$$

在证明定理之前,我们需要一个引理.

引理$\quad $设$A\in M_n (\mathbb{R} )$.则有关系式

$$a_{i1} A_{j1} +a_{i2} A_{j2} +\cdots +a_{in} A_{jn} =\delta _{ij} \det A,\label{1} \tag{1} $$

$$a_{1i} A_{1j} +a_{2i} A_{2j} +\cdots +a_{ni} A_{nj} =\delta _{ij} \det A,\label{2} \tag{2} $$

其中$\delta_{ij}$是克罗内克符号(当$i\neq j$时,$\eqref{1}$(或$\eqref{2}$)叫作行列式$\det A$按照错行(或错列)展开).

证明$\quad $当$i=j $时,引理的结论与$\S 2$定理$1$一致.因而只需考察$i\neq j$,$\delta _{ij} =0$的情况.为此引入矩阵

$$A’=[A_{(1)} ,\cdots ,A_{(i)} ,\cdots ,A_{(i)} ,\cdots ,A_{(n)} ]=\begin{pmatrix} a_{11} & a_{12} & \cdots & a_{1n} \\ \cdots & \cdots & \cdots & \cdots \\ a_{i1} & a_{i2} & \cdots & a_{in} \\ \cdots & \cdots & \cdots & \cdots \\ a_{i1} & a_{i2} & \cdots & a_{in} \\ \cdots & \cdots & \cdots & \cdots \\ a_{n1} & a_{n2} & \cdots & a_{nn} \end{pmatrix} ,$$

它是将矩阵$A=[\cdots ,A_{(i)} ,\cdots ,A_{(j)} ,\cdots ]$的第$j$行换成第$i$行得到的(第$i$行保留不动).如同所有带有两个相同行的方阵,$\det A’=0$.另一方面,代数余子式$A’_{jk} (k=1,\cdots ,n)$,是用划去$A’$的第$j$行$A’_{(j)} =A_{(i)}$和第$k$列得到的,因此$A’_{jk} =A_{jk}$.将行列式$A’=(a’_{st} )$按照第$j$行展开,得到关系式

$$0=\det A’=\sum_{k=1}^n a’_{jk} A’_{jk} =\sum_{k=1}^n a_{ik} A_{jk} ,$$

恰为引理中的$\eqref{1}$式.$\eqref{2}$式可由关于列的类似性质得到.

转入定理的证明,我们只要注意到$\eqref{1}$式的左边是矩阵$C=AA^{\vee }$的元素$c_{ij}$:

$$\begin{pmatrix} c_{11} & \cdots & c_{1n} \\ \cdots & \cdots & \cdots \\ c_{n1} & \cdots & c_{nn} \end{pmatrix} = \begin{pmatrix} a_{11} & \cdots & a_{1n} \\ \cdots & \cdots & \cdots \\ a_{n1} & \cdots & a_{nn} \end{pmatrix} \begin{pmatrix} A_{11} & \cdots & A_{n1} \\ \cdots & \cdots & \cdots \\ A_{1n} & \cdots & A_{nn} \end{pmatrix} .$$

根据$\eqref{1}$式,$(c_{ij} )=(\delta _{ij} \det A) =(\det A)E$.于是

$$AA^{\vee } =(\det A)E,$$

当$\det A\neq 0$时,我们得到

$$(\det A)^{-1} (AA^{\vee }) =A(\det A)^{-1} A^{\vee } =E.$$

$\eqref{2}$式的左边是矩阵$C’= A^{\vee } A$的元素$c’_{ij}$的表达式.因为$\eqref{1}$和$\eqref{2}$的右边是相同的,故当$\det A\neq 0$时,我们得到

$$A(\det A)^{-1} A^{\vee } =(\det A)^{-1} A^{\vee } A =E,$$

于是$A^{-1} =(\det A)^{-1} A^{\vee }$.

推论$\quad $一个行列式等于零,当且仅当它的行(或列)线性相关.

证明$\quad $矩阵$A\in M_n (\mathbb{R} )$的行(或列)的线性相关性,等价于$\text{rank} A < n$,即矩阵$A$的退化性,根据定理$1$,等价于条件$\det A=0$.

注记$\quad \text{rank} A < n \Rightarrow \det A=0$实际上是行列式基本性质的直接推论(见$\S 2,\text{D2} ,\text{D6}$).

定理$1$的理论价值(比实用价值)大.从计算的角度来看,特别是当矩阵的阶数较大时,求逆矩阵$A^{-1}$用第$2$章$\S 7$给出的$(P,Q)$算法更方便一些.

克拉默公式

现在我们来推导含$n$个未知数,$n$个方程的线性方程组的求解公式,行列式理论最初的发展就是为了这件事情.

定理2(克拉默)$\quad $如果线性方程组

$$\begin{matrix} a_{11} x_1 +\cdots +a_{1n} x_n =b_1 , \\ \cdots \cdots \cdots \\ a_{n1} x_1 +\cdots +a_{nn} x_n =b_n \end{matrix} $$

的系数行列式非零(即$\det (a_{ij} ) \neq 0$),则它有下述唯一解:

$$x_k^0 =\dfrac{\begin{vmatrix} a_{11} & \cdots & b_1 & \cdots & a_{1n} \\ \cdots & \cdots & \cdots & \cdots & \cdots \\ a_{n1} & \cdots & b_n & \cdots & a_{nn} \end{vmatrix}}{\begin{vmatrix} a_{11} & \cdots & a_{1k} & \cdots & a_{1n} \\ \cdots & \cdots & \cdots & \cdots & \cdots \\ a_{n1} & \cdots & a_{nk} & \cdots & a_{nn} \end{vmatrix} } ,k=1,2,\cdots ,n $$

(其中分子$D_k$是用自由项组成的列替换$D=\det (a_{ij} )$中的第$k$列得到的).

证明$\quad $根据定理$1$,矩阵$A=(a_{ij} )$可逆.因此将我们的方程组写成$AX=B$的形式,如同第$2$章$\S 3$第$8$段那样,我们得到

$$\begin{pmatrix} x_1^0 \\ \vdots \\ x_k^0 \\ \vdots \\ x_n^0 \end{pmatrix} =A^{-1}B =\dfrac{1}{\det A} \begin{pmatrix} A_{11} & A_{21} & \cdots & A_{n1} \\ A_{12} & A_{22} & \cdots & A_{n2} \\ \cdots & \cdots & \cdots & \cdots \\ A_{1n} & A_{2n} & \cdots & A_{nn} \end{pmatrix} \begin{pmatrix} b_1 \\ b_2 \\ \vdots \\ b_n \end{pmatrix} ,$$

从而

$$x_k^0 =\dfrac{1}{\det A} \sum_{i=1}^n A_{ik} b_i =\dfrac{1}{\det A} (b_1 A_{1k} +b_2 A_{2k} +\cdots + b_n A_{nk} ),k=1,2,\cdots ,n.$$

其中分子的表达式恰为行列式$D_k $按照第$k$列的展开式(见公式$\eqref{2}$).

将上述的全部过程反推回去,可以看到$\left( \dfrac{D_1 }{\det A} ,\cdots ,\dfrac{D_n }{\det A} \right) $确实是方程组的解.

注意到第$1$章$\S 4$的公式$(3)$、$(9)$分别与$n=2$和$n=3$时的克拉默公式一致.当$n$较小时,用克拉默公式(解线性方程组)是方便的,在一般情况下,可以把克拉默公式看作纯理论函数.例如将其用于在第$1$章$\S 3$第$5$段中给出的线性方程组,就得到斐波那契数的表达式(考虑到$\det A=1 $)

$$f_n =\begin{vmatrix} 1 & 0 & 0 & \cdots & 0 & 0 & 1 \\ 0 & 1 & 0 & \cdots & 0 & 0 & 1 \\ -1 & -1 & 1 & \cdots & 0 & 0 & 0 \\ \cdots & \cdots & \cdots & \cdots & \cdots & \cdots & \cdots \\ 0 & 0 & 0 & \cdots & -1 & 1 & 0 \\ 0 & 0 & 0 & \cdots & -1 & -1 & 0 \end{vmatrix} .$$

显然它比我们在第$2$章$\S 3$第$5$段中求出的关于$f_n $的清晰的表达式相差甚远.

还应当指出,使用克拉默公式的不可或缺的条件$\det A\neq 0$有时是不稳定的.对于实用中带有近似系数的线性方程组,计算精确度的提高可能会使情况发生根本的改变.例如

$$A_{\varepsilon } =\begin{pmatrix} -1 & 10 & 0 & \cdots & 0 & 0 \\ 0 & -1 & 10 & \cdots & 0 & 0 \\ \cdots & \cdots & \cdots & \cdots & \cdots & \cdots \\ 0 & 0 & 0 & \cdots & -1 & 10 \\ \varepsilon & 0 & 0 & \cdots & 0 & -1 \end{pmatrix} \in M_{10} (\mathbb{R} ),$$

则$\det A_{\varepsilon } =1-\varepsilon \cdot 10^9$(将行列式按照第一列的元素展开).当$\varepsilon =10^{-9}$时,有$\det A_{\varepsilon } =0$.同时,若系数矩阵的计算仅仅精确到百万分之一,我们可以“忽略$\varepsilon $”(即令$\varepsilon =0$,则$\det A=1$).由此看来,克拉默公式的适用条件对系数系统的微小“扰动”是敏感的.

加边子式法

第$2$章$\S 3$包含了求长方形线性方程组解集的全部所需资料.在这一过程中,矩阵的秩的概念至关重要.我们仅需将其转化为行列式理论的语言,就可以得到计算矩阵秩的另一种方法以及判断线性空间$\mathbb{R} ^m$中的向量组线性无关性的便利手段.

$$A=\begin{pmatrix} a_{11} & \cdots & a_{1r} & \cdots & a_{1n} \\ \cdots & \cdots & \cdots & \cdots & \cdots \\ a_{r1} & \cdots & a_{rr} & \cdots & a_{rn} \\ \cdots & \cdots & \cdots & \cdots & \cdots \\ a_{m1} & \cdots & a_{mr} & \cdots & a_{mn} \end{pmatrix} $$

是任意一个$m\times n$阶长方矩阵,其系数$a_{ij} \in \mathbb{R}$.

定义$\quad m\times n$矩阵$A$的任意$k$行和$k$列($k\leq \min{(m,n)}$)交叉处的元素组成一个方阵,其行列式叫作$A$的一个$k$阶子式.如果$i_1 ,\cdots ,i_k $和$j_1 ,\cdots ,j_k $分别是行指标和列指标,子式记作

$$M\begin{pmatrix} i_1 & \cdots & i_k \\ j_1 & \cdots & j_k \end{pmatrix} $$

当$m=n$且$k=n-1$时,我们就得到了先前引入的$n\times n$矩阵$A$的子式$M_{ij}$.

子式$\widetilde{M}$叫作$M$的加边,若$M$是由$\widetilde{M}$去掉一端的行(第一行或最后一行),以及一端的列得到的.

定理(加边子式法)$\quad $在计算矩阵$A$的秩时,可由$A$中较小阶的子式转到较大阶的子式.如果已找到$A$的一个$r$阶子式$M\neq 0$,那么仅需计算$r+1$阶子式,即子式$M$的加边.如果它们都等于零,则$\text{rank} A=r$.

证明$\quad $论断基于一个简单的事实,如果矩阵$A$的全部$k$阶子式等于零,则任意更高阶的子式亦等于零.为此,根据$\S 2$定理$1$,观察任意$(k+1)$阶子式按照任意列的元素的展开式(例如第一或最后一列,如果仅限于考察借助加边得到的子式),然后进入$(k+2)$阶子式,等等.

按照定理叙述中所指出的模式,设我们已达到某个$r$阶子式$M\neq 0$.不失一般性,假设与$M$相应的矩阵位于矩阵$A$的左上角:

$$A=\begin{vmatrix} \begin{array}{ccc|} a_{11} & \cdots & a_{1r} \\ & M & \\ a_{r1} & \cdots & a_{rr} \\ \hline \end{array}
& \begin{matrix} \cdots & a_{1j} & \cdots & a_{1n} \\ & \cdots & \cdots & \cdots \\ \cdots & a_{rj} & \cdots & a_{rn} \end{matrix} \\ \begin{matrix} \cdots & \cdots & \cdots \\ a_{i1} & \cdots & a_{ir} \\ \cdots & \cdots & \cdots \\ a_{m1} & \cdots & a_{mr} \end{matrix} & \begin{matrix} \cdots & \cdots & \cdots & \cdots \\ \cdots & a_{ij} & \cdots & a_{in} \\ \cdots & \cdots & \cdots & \cdots \\ \cdots & a_{mj} & \cdots & a_{mn} \end{matrix} \end{vmatrix} .$$

这件事总可以通过交换矩阵的行与列做到,自然地,这种作法不改变矩阵$A$的秩.

现在在$A$中取出任意一行$A_{(i)}$和任意一列$A^{(j)}$(可能$i\leq r$或$j\leq r$).在$A_{(i)}$和$A^{(j)}$的元素的帮助下我们组成了一个$r+1$阶子式$\widetilde{M}$,它是$M$的加边:

$$\widetilde{M} =\begin{vmatrix} a_{11} & \cdots & a_{1r} & a_{1j} \\ \cdots & \cdots & \cdots & \cdots \\ a_{r1} & \cdots & a_{rr} & a_{rj} \\ a_{i1} & \cdots & a_{ir} & a_{ij} \end{vmatrix} .$$

如果$\widetilde{M} \neq 0$,我们再考虑$\widetilde{M}$的加边.当$M$的全部加边子式为零时,我们就到达了临界时刻.

设对于任意$i,j,\widetilde{M} =0$.将$\widetilde{M}$按照最后一行展开,得到关系式

$$a_{i1} M_1 +a_{i2} M_2 +\cdots +a_{ir} M_r +a_{ij} M =0,$$

其中系数

$$M_s =(-1)^{r+s+1} \begin{vmatrix} a_{11} & \cdots & a_{1,s-1} & a_{1,s+1} & \cdots & a_{1r} & a_{1j} \\ \cdots & \cdots & \cdots & \cdots & \cdots & \cdots & \cdots \\ a_{r1} & \cdots & a_{r,s-1} & a_{r,s+1} & \cdots & a_{rr} & a_{rj} \end{vmatrix} ,$$

不依赖于$i$的选择.因为$M\neq 0$,故

$$a_{ij} =\lambda _1 a_{i1} +\lambda _2 a_{i2} +\cdots +\lambda _r a_{ir} ,$$

其中$\lambda _s =\dfrac{-M_s }{M} ,1\leq s\leq r$,此式对$i=1,2,\cdots ,m$均成立.于是

$$A^{(j)} =\lambda _1 A^{(1)} +\lambda _2 A^{(2)} +\cdots +\lambda _r A^{(r)} ,$$

即矩阵$A$的任意一列是前$r$列的线性组合.这就表明$\text{rank} A\leq r$.但是从$M\neq 0$,得到$M$中的列是线性无关的,自然地,它们所对应的$A$当中更长的列也是线性无关的.我们得到了结论$\text{rank} A =r$.

推论$\quad $任意矩阵的秩等于它的非零子式的最大阶数.

推论亦可独立于定理进行证明.设矩阵$A$的秩等于$r$.根据第$2$章$\S 2 $定理$1$,$r$是矩阵$A$的线性无关行的最大个数,也是线性无关列的最大个数.运用第$2$章$\S 3$定理$6$,我们发现,

$$A=B\begin{pmatrix} E_r & 0 \\ 0 & 0 \end{pmatrix} C,$$

其中$B$和$C$分别是非退化的$m$阶和$n$阶方阵,可以写成初等矩阵的乘积.因为矩阵$\begin{pmatrix} E_r & 0 \\ 0 & 0 \end{pmatrix}$有一个非零$r$阶子式$M=\vert E_r \vert =1$,但是没有阶数$> r$的非零子式,又因为这一性质在行与列的初等变换下保持不变,我们就得到了所需的论断.

加边子式法是相当实用的,特别是当我们不仅想知道秩,也想知道矩阵$A$的行或列的极大线性无关组的时候.然而在初等变换下这一信息是会失掉的.

习题

$1$.证明下述公式:

$$(AB)^{\vee } =B^{\vee }A^{\vee } ; {\sideset{^t}{}A}^{\vee } =\sideset{^t}{}(A^{\vee });$$

$$(\lambda A)^{\vee } =\lambda ^{n-1} A^{\vee } ;(A^{\vee })^{\vee } =(\det A)^{n-2} A.$$

证明:$(1)\;$证一:

$$\begin{align}
& (AB)^{\vee }(i;j) \begin{pmatrix} 1,\cdots ,j-1,j+1,\cdots ,n \\ 1,\cdots ,i-1,i+1,\cdots ,n \end{pmatrix} \\
= & (-1)^{i+j} \sum_{1\leq v_1 < v_2 < \cdots < v_{n-1} \leq n} A\begin{pmatrix} 1,\cdots ,j-1,j+1,\cdots ,n \\ v_1 ,v_2 ,\cdots ,v_{n-1} \end{pmatrix} B\begin{pmatrix} v_1 ,v_2 ,\cdots ,v_{n-1} \\ 1,\cdots ,i-1,i+1,\cdots ,n \end{pmatrix} \\
= & (-1)^{i+j} \sum_{k=1}^n A\begin{pmatrix} 1,\cdots ,j-1,j+1,\cdots ,n \\ 1,\cdots ,k-1,k+1,\cdots ,n \end{pmatrix} B\begin{pmatrix} 1,\cdots ,k-1,k+1,\cdots ,n \\ 1,\cdots ,i-1,i+1,\cdots ,n \end{pmatrix} \\
= & \sum_{k=1}^n (-1)^{j+k} A\begin{pmatrix} 1,\cdots ,j-1,j+1,\cdots ,n \\ 1,\cdots ,k-1,k+1,\cdots ,n \end{pmatrix} (-1)^{k+i} B\begin{pmatrix} 1,\cdots ,k-1,k+1,\cdots ,n \\ 1,\cdots ,i-1,i+1,\cdots ,n \end{pmatrix} \\
= & \sum_{k=1}^n A_{jk} B_{ki} \\
= & \sum_{k=1}^n A^{\vee }(k;j) B^{\vee }(i;k) \\
= & B^{\vee } A^{\vee }(i;j) .\\
\end{align}$$

因此

$$(AB)^{\vee } =B^{\vee }A^{\vee } .$$

证二:Ⅰ.当$\det (AB) \neq 0$时,这时$\det A \neq 0$,$\det B\neq 0$,由公式$A^{\vee } =(\det A) A^{-1} $,可得

$$(AB)^{\vee } =(\det AB) (AB)^{-1} =(\det B)B^{-1} (\det A) A^{-1} =B^{\vee }A^{\vee } .$$

Ⅱ.当$\det AB =0$时,考虑矩阵$A(\lambda )=A-\lambda E,B(\lambda )=B-\lambda E$,由于$A$和$B$都最多只有有限个特征值,因为存在无穷多个$\lambda $,使

$$\det A(\lambda ) \neq 0,\det B(\lambda ) \neq 0,\label{3} \tag{3} $$

那么由Ⅰ的结论有

$$(A(\lambda )B(\lambda ))^{\vee } =(B(\lambda ))^{\vee } (A(\lambda ))^{\vee }.\label{4} \tag{4} $$

令$(A(\lambda )B(\lambda ))^{\vee } =(f_{ij} (\lambda ))_{n\times n} ,(B(\lambda ))^{\vee } (A(\lambda ))^{\vee } =(g_{ij} (\lambda ))_{n\times n}$,则由$\eqref{4}$有

$$f_{ij} (\lambda )=g_{ij} (\lambda ),(i,j=1,2,\cdots ,n)\label{5} \tag{5} $$

由于有无穷多个$\lambda $使$\eqref{3}$式成立,从而有无穷多个$\lambda $使$\eqref{5}$式成立,但$f_{ij} (\lambda ) ,g_{ij} (\lambda )$都是多项式,从而$\eqref{5}$式对一切$\lambda $都成立.特别令$\lambda =0$,这时有

$$(AB)^{\vee } =(A(0)B(0))^{\vee }=(B(0))^{\vee } (A(0))^{\vee } =B^{\vee }A^{\vee } .$$

$(2)\;$设$A=(a_{ij} )_{n\times n}$,则

$$\sideset{^t}{}A=\begin{pmatrix} a_{11} & a_{21} & \cdots & a_{n1} \\ a_{12} & a_{22} & \cdots & a_{n2} \\ \cdots & \cdots & \cdots & \cdots \\ a_{1n} & a_{2n} & \cdots & a_{nn} \end{pmatrix} ,$$

$$A^{\vee }=\begin{pmatrix} A_{11} & A_{21} & \cdots & A_{n1} \\ A_{12} & A_{22} & \cdots & A_{n2} \\ \cdots & \cdots & \cdots & \cdots \\ A_{1n} & A_{2n} & \cdots & A_{nn} \end{pmatrix} ,$$

于是,

$$(\sideset{^t}{}A)^{\vee }=\begin{pmatrix} A_{11} & A_{12} & \cdots & A_{1n} \\ A_{21} & A_{22} & \cdots & A_{2n} \\ \cdots & \cdots & \cdots & \cdots \\ A_{n1} & A_{n2} & \cdots & A_{nn} \end{pmatrix} ,$$

$$\sideset{^t}{}(A^{\vee })=\begin{pmatrix} A_{11} & A_{12} & \cdots & A_{1n} \\ A_{21} & A_{22} & \cdots & A_{2n} \\ \cdots & \cdots & \cdots & \cdots \\ A_{n1} & A_{n2} & \cdots & A_{nn} \end{pmatrix} ,$$

故${\sideset{^t}{}A}^{\vee } =\sideset{^t}{}(A^{\vee }).$

$(3)\;$设$A=(a_{ij} )_{n\times n}$,再设$(\lambda A)^{\vee } =(b_{ij} )_{n\times n}$,那么$b_{ij}$为矩阵$\lambda A$划去第$j$行和第$i$列的代数余子式($n-1$阶行列式),其中每行提出公因子$\lambda $后,可得

$$b_{ij} =\lambda ^{n-1} A_{ji} \quad (i,j=1,2,\cdots ,n)$$

由此即证$(\lambda A)^{\vee } =\lambda ^{n-1} A^{\vee }$.

$(4)\;$Ⅰ.当$n\geq 3$时.

若$\det A\neq 0$,则$\det A^{\vee } =(\det A)^{n-1} $.由于

$$A^{\vee } (A^{\vee })^{\vee } =(\det A^{\vee })E.$$

因此$(A^{\vee })^{\vee } =(\det A^{\vee })(A^{\vee })^{-1} =(\det A)^{n-1} (\dfrac{1}{\det A } A)=(\det A)^{n-2} A$.

若$\det A=0$,则据习题$2$的结果得,

$$\text{rank} A^{\vee } \leq 1 < n-1$$

因此,$(A^{\vee })^{\vee } =0$.于是结论也成立.

Ⅱ.设$n=2$.此时

$$A=\begin{pmatrix} a & b \\ c & d \end{pmatrix} $$

从而

$$A^{\vee }=\begin{pmatrix} d & -b \\ -c & a \end{pmatrix} .$$

因此

$$(A^{\vee })^{\vee }=\begin{pmatrix} a & b \\ c & d \end{pmatrix} =A.$$

由Ⅰ、Ⅱ可知,若$A$是$n$级矩阵$(n\geq 2)$,则$(A^{\vee })^{\vee } =(\det A)^{n-2} A$.

$2$.通过$\text{rank} A$表示$\text{rank} A^{\vee }$.

证明:若$\text{rank} A=n$,则$\det A\neq 0$.由于

$$AA^{\vee } =(\det A)E$$

因此$\det A \det A^{\vee } =(\det A)^n \neq 0$.从而$\det A^{\vee } \neq 0$.于是$\text{rank} A^{\vee } =n$.

若$\text{rank} A=n-1$,则$A$有一个$n-1$阶子式不等于$0$.从而$A$有一个元素的代数余子式不等于$0$.于是$A^{\vee } \neq 0$.由于$\det A=0$,因此$AA^{\vee } =(\det A)E=0$.从而

$$\text{rank} A+\text{rank} A^{\vee } \leq n.$$

于是$\text{rank} A^{\vee } \leq n-\text{rank} A=n-(n-1) =1$.

由于$A^{\vee } \neq 0$,因此$\text{rank} A^{\vee } =1$

若$\text{rank} A < n-1$,则$A$的所有$n-1$阶子式都等于$0$,从而$A^{\vee } =0$.于是$\text{rank} A^{\vee }=0$.

因此,若$A$是$n$级矩阵$(n\geq 2)$,则

$$\text{rank} A^{\vee }=\begin{cases} n, & \text{rank} A=n, \\ 1, & \text{rank} A=n-1, \\ 0 ,& \text{rank} A\leq n-2 .\end{cases} $$

$3$.证明当未知量的个数与方程的个数相等时,齐次线性方程组有非零解,当且仅当系数矩阵的行列式等于零.

证明:设数域$K$上的$n$元齐次线性方程组

$$\begin{cases} a_{11} x_1 +\cdots a_{1n} x_n =0 \\ \cdots \cdots \cdots \cdots \cdots \cdots \cdots \\ a_{n1} x_1 +\cdots a_{nn} x_n =0 \end{cases} \label{6} \tag{6} $$

的系数矩阵为$A=(a_{ij} )$,增广矩阵为$\widetilde{A}$.对$\widetilde{A}$施行初等行变换化成行阶梯形矩阵$\widetilde{J}$,则相应地,$A$经过这些初等行变换被化成行阶梯形矩阵$J$,其中$J$比$\widetilde{J}$少最后一列.我们有$\vert J\vert =l\vert A\vert $,其中$l$是$K$中某个非零数.于是

$(1)\;$

$$\vert A\vert \neq 0 \Rightarrow \vert J\vert \neq 0\Rightarrow J没有零行\Rightarrow J有n个主元 $$

设$J$的$n$个主元的列指标依次为$j_1 ,j_2 ,\cdots ,j_n $,则$1\leq j_1 < j_2 < \cdots < j_n \leq n$.由此得$j_1 =1 ,j_2 =2 ,\cdots ,j_n =n $.因此

$$J=\begin{pmatrix} c_{11} & c_{12} & \cdots & c_{1n} \\ 0 & c_{22} & \cdots & c_{2n} \\ \cdots & \cdots & \cdots & \cdots \\ 0 & 0 & \cdots & c_{nn} \end{pmatrix} ,$$

其中$c_{11} \neq 0,c_{22} \neq 0,\cdots ,c_{nn} \neq 0$.从而

$$\widetilde{J} =\begin{pmatrix} c_{11} & c_{12} & \cdots & c_{1n} & 0 \\ 0 & c_{22} & \cdots & c_{2n} & 0 \\ \cdots & \cdots & \cdots & \cdots & \cdots \\ 0 & 0 & \cdots & c_{nn} & 0\end{pmatrix} ,$$

于是$\widetilde{J}$表示的阶梯形方程组不会出现方程“$0=d$”(其中$d$是非零数),因此当$\vert A\vert \neq 0$时,方程组$\eqref{6}$有且为零解;由于阶梯形方程组的方程个数等于$n$,因此$\eqref{6}$有唯一解.

$(2)\;$若$\vert A\vert =0$,则$\vert J\vert =0$.假如$J$没有零行,则从上面一段知道$\vert J\vert =c_{11} c_{22} \cdots c_{nn} \neq 0$,矛盾.因此$J$必有零行.从而

$$\widetilde{J} =\begin{pmatrix} c_{11} & \cdots & \cdots & \cdots & \cdots & \cdots & \cdots & \cdots & c_{1n} & 0 \\ 0 & \cdots & 0 & c_{2j_2 } & \cdots & \cdots & \cdots & \cdots & c_{2n} & 0 \\ \cdots & \cdots & \cdots & \cdots & \cdots & \cdots & \cdots & \cdots & \cdots & \cdots \\ 0 & \cdots & \cdots & \cdots & \cdots & 0 & c_{rj_r } & \cdots & c_{rn} & 0 \\ 0 & \cdots & \cdots & \cdots & \cdots & \cdots & \cdots & \cdots & 0 & 0 \\ \cdots & \cdots & \cdots & \cdots & \cdots & \cdots & \cdots & \cdots & \cdots & \cdots \\ 0 & \cdots & \cdots & \cdots & \cdots & \cdots & \cdots & \cdots & 0 & 0 \end{pmatrix} ,$$

其中$c_{11} \neq 0,c_{2j_2 } \neq 0,\cdots ,c_{rj_r } \neq 0;r < n$.

此时,因为位于矩阵$\widetilde{J}$第$r+1$行、第$n+1$列的数为$0$,故方程组$\eqref{6}$有无穷多个非零解.

结合$(1),(2)$的讨论,当未知量的个数与方程的个数相等时,齐次线性方程组有非零解,当且仅当系数矩阵的行列式等于零.

$4$.运用第$2$章$\S 3$第$8$段的结果和定理$2$证明,当秩$r=n-1$时,齐次线性方程组

$$\begin{align}
& a_{11} x_1 +\cdots +a_{1n} x_n =0, \\
&\cdots \cdots \cdots \cdots \cdots \cdots \cdots \cdots \\
& a_{n-1,1} x_1 +\cdots +a_{n-1,n} x_n =0\end{align}$$

的基础解系由一个列向量

$$X^0 =[D_1 ,-D_2 ,D_3 ,\cdots ,(-1)^{n-1} D_n ]$$

组成,其中$D_i$是从$A=(a_{ij})$中去掉第$i$列所得矩阵的行列式.任意解的形式为$X=\lambda X^0 $.

证明:在所给它的齐次线性方程组的下面添上一个方程:

$$0x_1 +0x_2 +\cdots +0x_n =0.$$

得到$n$个方程的$n$元齐次线性方程组,其系数矩阵$B=\begin{pmatrix} A \\ 0 \end{pmatrix}$.$B$的$(n,i)$元的代数余子式$A_{ni}$为

$$A_{ni} =(-1)^{n+i} D_i ,i=1,2,\cdots ,n.$$

原齐次线性方程组的第$j$个方程($j=1,2,\cdots ,n-1$)为

$$a_{j1} x_1 +a_{j2} x_2 +\cdots +a_{jn} x_n =0.$$

对$\vert B\vert $用行列式按一行展开定理,得

$$a_{j1} A_{n1} +a_{j2} A_{n2} +\cdots +a_{jn} A_{nn} =0,j=1,2,\cdots ,n-1.$$

由此得出,$X^{0}=[D_1 ,-D_2 ,D_3 ,\cdots ,(-1)^{n-1} D_n ]$是原齐次线性方程组的一个解.

因为$\text{rank} A=n-1$,故$X^{0} $是原齐次线性方程组的一个非零解.任意解的形式为$X=\lambda X^0 $.

齐次线性方程组的解空间$W$的维数为

$$\text{dim} W=n-\text{rank} A=n-(n-1)=1.$$

因此$X^{0}$是$W$的一个基,即$X^{0}$是原齐次线性方程组的一个基础解系.任意解的形式为$X=\lambda X^0 $.

$5$.设$A=(a_{ij}) \in M_n (\mathbb{R} )$,且任取$i\neq j$,有$(n-1)\vert a_{ij} \vert < \vert a_{ii} \vert$.证明$\det A\neq 0$.

提示:假设命题不真,利用习题$3$给出的准则.即如果$[x_1^0 ,\cdots ,x_n^0 ]$是线性方程组$AX=0$的非平凡解,且$x_k^0$是它的具有极大模的分量,则从第$k$个方程

$$a_{kk} x_k^0 +\sum_{j\neq k} a_{kj} x_j^0 =0$$

得到的估值

$$(n-1)\vert a_{kk} \vert \vert x_k^0 \vert =(n-1) \left| \sum_{j\neq k} a_{kj} x_j^0 \right| < (n-1) \vert a_{kk} \vert \vert x_k^0 \vert ,$$

得到矛盾.

证明:对于任意的$i\neq j $,有$(n-1)\vert a_{ij} \vert < \vert a_{ii} \vert$成立,即

$$(n-1)\vert a_{i1} \vert < \vert a_{ii} \vert ,$$

$$\vdots $$

$$(n-1)\vert a_{i,i-1} \vert < \vert a_{ii} \vert ,$$

$$(n-1)\vert a_{i,i+1} \vert < \vert a_{ii} \vert ,$$

$$\vdots $$

$$(n-1)\vert a_{in} \vert < \vert a_{ii} \vert .$$

把上面的$n-1$个式子相加得,

$$(n-1)\sum_{j=1,j\neq i}^n \vert a_{ij} \vert < (n-1)\vert a_{ii} \vert ,$$

$$\sum_{j=1,j\neq i}^n \vert a_{ij} \vert < \vert a_{ii} \vert .$$

下面开始证明$\det A\neq 0$.先证矩阵$A$的列向量组$\alpha _1 ,\alpha _2 ,\cdots ,\alpha _n $线性无关.

假如$\alpha _1 ,\alpha _2 ,\cdots ,\alpha _n $线性相关,则有一组不全为$0$的数$k_1 ,k_2 ,\cdots ,k_n $,使得

$$k_1 \alpha _1 +k_2 \alpha _2 +\cdots +k_n \alpha _n =0 .\label{7} \tag{7} $$

不妨设

$$\vert k_l \vert =\max{\lbrace \vert k_1 \vert ,\vert k_2 \vert ,\cdots ,\vert k_n \vert \rbrace } $$

在$\eqref{7}$式考虑第$l$个分量的等式:

$$k_1 a_{l1} +k_2 a_{l2} +\cdots +k_l a_{ll} +\cdots +k_n a_{ln} =0.\label{8} \tag{8} $$

从$\eqref{8}$式得

$$\begin{align}
a_{ll} & =-\dfrac{k_1 }{k_l } a_{l1} -\cdots -\dfrac{k_{l-1} }{k_l } a_{l,l-1} -\dfrac{k_{l+1} }{k_l } a_{l,l+1} -\cdots -\dfrac{k_n }{k_l } a_{ln} \\
& =-\sum_{j=1,j\neq l}^n \dfrac{k_j }{k_l } a_{lj} . \label{9} \tag{9}
\end{align}$$

从$\eqref{9}$式得

$$\vert a_{ll} \vert \leq \sum_{j=1,j\neq l}^n \dfrac{\vert k_j \vert }{\vert k_l \vert } \vert a_{lj} \vert \leq \sum_{j=1,j\neq l}^n \vert a_{lj} \vert .$$

这与已知条件矛盾.因此$\alpha _1 ,\alpha _2 ,\cdots ,\alpha _n $线性无关,从而它的秩等于$n$,亦即表明矩阵$A$的行列式$\det A\neq 0$.

$6$.证明下述论断(比内-柯西定理)设$A=(a_{ij}) ,B=(b_{kl})$分别是$n\times m$和$m\times n$阶矩阵,且$C=AB$.则

$$\det C=\sum_{1\leq j_1 \leq \cdots \leq j_n \leq m} \begin{vmatrix} a_{1 j_1 } & a_{2 j_1 } & \cdots & a_{n j_1 } \\ a_{1 j_2 } & a_{2 j_2 } & \cdots & a_{n j_2 } \\ \cdots & \cdots & \cdots & \cdots \\ a_{1 j_n } & a_{2 j_n } & \cdots & a_{n j_n } \end{vmatrix} \times \begin{vmatrix} b_{j_1 1} & b_{j_1 2} & \cdots & b_{j_1 n} \\ b_{j_2 1} & b_{j_2 2} & \cdots & b_{j_2 n} \\ \cdots & \cdots & \cdots & \cdots \\ b_{j_n 1} & b_{j_n 2} & \cdots & b_{j_n n} \end{vmatrix} .$$

右边的和式遍历从$1,2,\cdots ,m$中取$n$个元素$\lbrace j_1 ,j_2 ,\cdots ,j_n \rbrace $的所有可能的$\displaystyle {m \choose n}$种组合.特别地,当$m=n$时,$\det C=\det A \cdot \det B$,而当$n > m $时,$\det C=0$.

提示:因为

$$C=(c_{ij}) ,c_{ij} =\sum_{k=1}^m a_{ik} b_{kj} ,$$

反复运用行列式的性质$\text{D2}$($\S 1$定理$2$)得到

$$\det C=\sum_{k_1 ,\cdots ,k_n =1}^m \begin{vmatrix} a_{1k_1 } & a_{1k_2 } & \cdots & a_{1k_n } \\ a_{2k_1 } & a_{2k_2 } & \cdots & a_{2k_n } \\ \cdots & \cdots & \cdots & \cdots \\ a_{nk_1 } & a_{nk_2 } & \cdots & a_{nk_n } \end{vmatrix} b_{k_1 1} b_{k_2 2} \cdots b_{k_n n} ,$$

其中求和遍历所有不同的$k_1 ,\cdots ,k_n $.当$m < n$时,没有这样的下标,故$\det C=0$.如果$m\geq n$,则$k_1 ,\cdots ,k_n $就是从$1,2,\cdots ,m$所取的元素$\lbrace j_1 ,\cdots ,j_n \rbrace $按某种顺序的排列,将对应于给定组合$\lbrace j_1 ,\cdots ,j_n \rbrace $的所有的项收集到一起,并在$\S 1$公式$(3)$的帮助下就可以得到所需的表达式:

$$\begin{align}
& \sum \begin{vmatrix} a_{1k_1 } & \cdots & a_{nk_1 } \\ \cdots & \cdots & \cdots \\ a_{1k_n } & \cdots & a_{nk_n } \end{vmatrix} b_{k_1 1} \cdots b_{k_n n} \\
= & \begin{vmatrix} a_{1j_1 } & \cdots & a_{nj_1 } \\ \cdots & \cdots & \cdots \\ a_{1j_n } & \cdots & a_{nj_n } \end{vmatrix} \sum \varepsilon_{\pi } b_{k_1 1} \cdots b_{k_n n} \\
= & \begin{vmatrix} a_{1j_1 } & \cdots & a_{1j_n } \\ \cdots & \cdots & \cdots \\ a_{nj_1 } & \cdots & a_{nj_n } \end{vmatrix} \cdot \begin{vmatrix} b_{j_1 1} & \cdots & b_{j_1 n} \\ \cdots & \cdots & \cdots \\ b_{j_n 1} & \cdots & b_{j_n n} \end{vmatrix} ,
\end{align}$$

其中$\begin{pmatrix} j_1 & \cdots & j_n \\ \cdots & \cdots & \cdots \\ k_1 & \cdots & k_n \end{pmatrix}$.

证明:如果$n > m$,则$\text{rank} (AB)\leq \text{rank} (A)\leq m < n$.所以$AB$不是满秩矩阵.从而$\det C=0$.

下面设$n\leq m$.考虑分块矩阵

$$\begin{pmatrix} A & 0 \\ -E_m & B \end{pmatrix} \label{10} \tag{10} $$

把它的第$2$块行的$A$倍加到第$1$块行上,即

$$\begin{pmatrix} E_n & A \\ 0 & E_m \end{pmatrix} \begin{pmatrix} A & 0 \\ -E_m & B \end{pmatrix} =\begin{pmatrix} 0 & AB \\ -E_m & B \end{pmatrix}\label{11} \tag{11} $$

利用$\det (AB) =(\det A)\cdot (\det B) $得,$\eqref{11}$式左端的矩阵的行列式等于公块矩阵$\eqref{10}$的行列式.为了计算$\eqref{11}$式右端矩阵的行列式,按前$n$行展开得

$$\begin{align}
\det \begin{pmatrix} 0 & AB \\ -E_m & B \end{pmatrix} & =\det (AB) (-1)^{(1+\cdots +n)+[(m+1)+\cdots +(m+s)]} \det (-E_m) \\
& =(-1)^{m(n+1)} \det (AB) \label{12} \tag{12}
\end{align}$$

我们来计算矩阵$\eqref{10}$的行列式,按前$n$行展开得

$$\begin{align}
\det \begin{pmatrix} A & 0 \\ -E_m & B \end{pmatrix} & =\sum_{1\leq j_1 < \cdots j_n \leq m} A\begin{pmatrix} 1, & 2, & \cdots , & n \\ j_1, & j_2, & \cdots , & j_n \end{pmatrix} \\
& \cdot (-1)^{(1+2+\cdots +n)+(j_1 +j_2 +\cdots +j_n )} \cdot \det (-\varepsilon _{\mu _1 } ,\cdots ,-\varepsilon _{\mu _{m-n} } \;,B) \label{13} \tag{13}
\end{align}$$

其中$\lbrace \mu _1 ,\mu _2 ,\cdots ,\mu _{m-n} \rbrace =\lbrace 1,2,\cdots ,m \rbrace \setminus \lbrace j_1 ,j_2 ,\cdots ,j_n \rbrace$.把行列式$\det (-\varepsilon _{\mu _1 } ,\cdots ,-\varepsilon _{\mu _{m-n} } \;,B)$按前$m-n$列展开,注意前$m-n$列只有一个$m-n$级子式不为零,它是取第$\mu _1 ,\mu _2 ,\cdots ,\mu _{m-n}$行得到的那个$m-n$级子式,这个子式的代数余子式是

$$(-1)^{(\mu _1 +\mu _2 +\cdots +\mu _{m-n} )+[1+2+\cdots +(m-n)] } \cdot B\begin{pmatrix} j_1, & j_2, & \cdots , & j_n \\ 1, & 2, & \cdots , & n \end{pmatrix} $$

因此

$$\begin{align}
& \det (-\varepsilon _{\mu _1 } ,\cdots ,-\varepsilon _{\mu _{m-n} } \;,B) \\
= & \det (-E_{m-n} ) \cdot B\begin{pmatrix} j_1, & j_2, & \cdots , & j_n \\ 1, & 2, & \cdots , & n \end{pmatrix} \\
& \cdot (-1)^{(\mu _1 +\cdots +\mu _{m-n} ) +[1+2+\cdots +(m-n)]}\label{14} \tag{14}
\end{align}$$

代入$\eqref{13}$式,$(-1)$的指数经过计算化简,

$$\begin{align}
& \det \begin{pmatrix} A & 0 \\ -E_m & B \end{pmatrix} \\
= & (-1)^{n^2 +m^2 -n(m+1)} \\
& \cdot \sum_{1\leq j_1 < \cdots < j_n \leq m} A\begin{pmatrix} 1, & 2, & \cdots , & n \\ j_1, & j_2, & \cdots , & j_n \end{pmatrix} B\begin{pmatrix} j_1, & j_2, & \cdots , & j_n \\ 1, & 2, & \cdots , & n \end{pmatrix} \label{15} \tag{15} \\
\end{align}$$

从$\eqref{11}$,$\eqref{12}$,$\eqref{15}$式得

$$\begin{align}
\det (AB) & =(-1)^{m(n+1)} (-1)^{n^2+m^2-n(m+1)} \\
& \cdot \sum_{1\leq j_1 < \cdots < j_n \leq m} A\begin{pmatrix} 1, & 2, & \cdots , & n \\ j_1, & j_2, & \cdots , & j_n \end{pmatrix} B\begin{pmatrix} j_1, & j_2, & \cdots , & j_n \\ 1, & 2, & \cdots , & n \end{pmatrix} \\
= & \sum_{1\leq j_1 < \cdots < j_n \leq m} A\begin{pmatrix} 1, & 2, & \cdots , & n \\ j_1, & j_2, & \cdots , & j_n \end{pmatrix} B\begin{pmatrix} j_1, & j_2, & \cdots , & j_n \\ 1, & 2, & \cdots , & n \end{pmatrix} ,\\
\end{align}$$

即证

$$\det C=\sum_{1\leq j_1 \leq \cdots \leq j_n \leq m} \begin{vmatrix} a_{1 j_1 } & a_{2 j_1 } & \cdots & a_{n j_1 } \\ a_{1 j_2 } & a_{2 j_2 } & \cdots & a_{n j_2 } \\ \cdots & \cdots & \cdots & \cdots \\ a_{1 j_n } & a_{2 j_n } & \cdots & a_{n j_n } \end{vmatrix} \times \begin{vmatrix} b_{j_1 1} & b_{j_1 2} & \cdots & b_{j_1 n} \\ b_{j_2 1} & b_{j_2 2} & \cdots & b_{j_2 n} \\ \cdots & \cdots & \cdots & \cdots \\ b_{j_n 1} & b_{j_n 2} & \cdots & b_{j_n n} \end{vmatrix} .$$

右边的和式遍历从$1,2,\cdots ,m$中取$n$个元素$\lbrace j_1 ,j_2 ,\cdots ,j_n \rbrace $的所有可能的$\displaystyle {m \choose n}$种组合.

$7$.利用上题证明,如果$A$是$\mathbb{R} $上的$m\times n$矩阵,$m\geq n$,则

$$\det \sideset{^t}{}AA=\sum_M M^2 ,$$

其中$M$遍历矩阵$A$的全部$\displaystyle {m \choose n}$个$n$阶子式.

证明:因为$m\geq n$,故$\sideset{^t}{}AA$的行列式为一$n$级主子式,即

$$\begin{align}
& \det \sideset{^t}{}AA \\
= & \sideset{^t}{}AA \begin{pmatrix} 1, & 2, & \cdots , & n \\ 1, & 2, & \cdots , & n \end{pmatrix} \\
= & \sum_{1\leq j_1 < \cdots < j_n \leq m} \sideset{^t}{}A\begin{pmatrix} 1, & 2, & \cdots , & n \\ j_1, & j_2, & \cdots , & j_n \end{pmatrix} A\begin{pmatrix} j_1, & j_2, & \cdots , & j_n \\ 1, & 2, & \cdots , & n \end{pmatrix} \\
= & \sum_{1\leq j_1 < \cdots < j_n \leq m} \left[ A\begin{pmatrix} 1, & 2, & \cdots , & n \\ j_1, & j_2, & \cdots , & j_n \end{pmatrix} \right] ^2 \\
\end{align}$$

上述第二个等号成立是因为

$$\sideset{^t}{}[A(1,\cdots ,n;j_1 ,\cdots ,j_n )] =\sideset{^t}{}A (j_1 ,\cdots ,j_n ;1,\cdots ,n)$$

设$M$遍历矩阵$A$的全部$\displaystyle {m \choose n}$个$n$阶子式,则

$$\det \sideset{^t}{}AA=\sum_M M^2 .$$

$8$.给定$n\times n$矩阵$A=(a_{ij})$的一个$k$阶子式(见第$3$段的定义)

$$M\begin{pmatrix} i_1 & \cdots & i_k \\ j_1 & \cdots & j_k \end{pmatrix} $$

相应的$(n-k)$阶余子式$\bar{M} \begin{pmatrix} i_1 & \cdots & i_k \\ j_1 & \cdots & j_k \end{pmatrix}$是从$A$中去掉第$i_1 ,\cdots ,i_k $行,和第$j_1 ,\cdots ,j_k $列所得矩阵的行列式.表达式

$$A\begin{pmatrix} i_1 & \cdots & i_k \\ j_1 & \cdots & j_k \end{pmatrix} =(-1)^{s(M)} \bar{M} \begin{pmatrix} i_1 & \cdots & i_k \\ j_1 & \cdots & j_k \end{pmatrix} ,$$

$$s(M) =(i_1 +\cdots +i_k )+(j_1 +\cdots +j_k ),$$

称为$M\begin{pmatrix} i_1 & \cdots & i_k \\ j_1 & \cdots & j_k \end{pmatrix}$的代数余子式.当$k=n-1$时,我们回到通常代数余子式的定义.利用行列式按照第$i_1 ,\cdots ,i_k $行元素的一系列展开证明下述

定理(拉普拉斯)$\quad $在矩阵$A=(a_{ij})$中取下标为$i_1 ,\cdots ,i_k $的$k$行.则

$$\det A=\sum_{1\leq j_1 < \cdots < j_k \leq n} M\begin{pmatrix} i_1 & \cdots & i_k \\ j_1 & \cdots & j_k \end{pmatrix} \cdot A\begin{pmatrix} i_1 & \cdots & i_k \\ j_1 & \cdots & j_k \end{pmatrix} .\label{16} \tag{16} $$

任取正整数$n$,在两种特殊情况下,拉普拉斯定理是我们已知的:$1)\;k=1$;$2)\;A$有阶数为$(n-k)\times k$的一个零角.在未知的情况下,哪怕取$n=4,i_1 =1 ,i_2 =2 $对拉普拉斯定理的正确性作一说明也是有益的:

$$\begin{align}
\det A=& \begin{vmatrix} a_{11} & a_{12} \\ a_{21} & a_{22} \end{vmatrix} \cdot \begin{vmatrix} a_{33} & a_{34} \\ a_{43} & a_{44} \end{vmatrix} -\begin{vmatrix} a_{11} & a_{13} \\ a_{21} & a_{23} \end{vmatrix} \cdot \begin{vmatrix} a_{32} & a_{34} \\ a_{42} & a_{44} \end{vmatrix} \\
& +\begin{vmatrix} a_{11} & a_{14} \\ a_{21} & a_{24} \end{vmatrix} \cdot \begin{vmatrix} a_{32} & a_{33} \\ a_{42} & a_{43} \end{vmatrix} -\begin{vmatrix} a_{12} & a_{13} \\ a_{22} & a_{23} \end{vmatrix} \cdot \begin{vmatrix} a_{31} & a_{34} \\ a_{41} & a_{44} \end{vmatrix} \\
& -\begin{vmatrix} a_{12} & a_{14} \\ a_{22} & a_{24} \end{vmatrix} \cdot \begin{vmatrix} a_{31} & a_{33} \\ a_{41} & a_{43} \end{vmatrix} -\begin{vmatrix} a_{13} & a_{14} \\ a_{23} & a_{24} \end{vmatrix} \cdot \begin{vmatrix} a_{31} & a_{32} \\ a_{41} & a_{42} \end{vmatrix} .
\end{align}$$

证明:$A$中取定的$k$行元素组成的$k$级子式一共有$\displaystyle {n \choose k}$个(从$A$的$n$列中每次取出$k$列的组合数),因此$\eqref{16}$的右端有$\displaystyle {n \choose k}$个乘积项.在每个乘积项中,$k$级子式$M\begin{pmatrix} i_1 & \cdots & i_k \\ j_1 & \cdots & j_k \end{pmatrix}$有$k!$项;$\bar{M} \begin{pmatrix} i_1 & \cdots & i_k \\ j_1 & \cdots & j_k \end{pmatrix}$有$(n-k)!$项.因此$\eqref{16}$式右端总共有$\displaystyle {n \choose k} \cdot k!(n-k)!=n!$项.这$n!$项两两不同.如果我们能证明$\eqref{16}$右端的每一项都是$\det A$的一项,那么$\eqref{16}$的右端$n!$项的和就正好是$\det A$.现在任取$\eqref{16}$的右端的一项

$$(-1)^{\tau (\mu _1 \cdots \mu _k )} a_{i_1 \mu _1 } \cdots a_{i_k \mu _k } \cdot (-1) ^{(i_1 +\cdots +i_k )+(j_1 +\cdots +j_k )} \cdot (-1)^{\tau (\nu _1 \cdots \nu _{n-k} )} a_{i’_1 \nu _1 } \cdots a_{i’_{n-k} \;\nu _{n-k} } \label{17} \tag{17} $$

其中$\mu _1 \cdots \mu _k$是$j_1 ,\cdots ,j_k$的一个$k$元排列,$\nu _1 \cdots \nu _{n-k}$是$j’_1 ,\cdots ,j’_{n-k}$的一个$n-k$元排列.我们来证明$\eqref{17}$是$\det A$的一项.显然,$a_{i_1 \mu _1 } \cdots a_{i_k \mu _k } a_{i’_1 \nu _1 } \cdots a_{i’_{n-k} \;\nu _{n-k} }$是$A$中不同行、不同列的$n$个元素的乘积,它是$\det A$的一项当且仅当它带的符号为

$$\begin{align}
& (-1)^{\tau (i_1 \cdots i_k i’_1 \cdots i’_{n-k} ) +\tau (\mu _1 \cdots \mu_k \nu _1 \cdots \nu _{n-k} )} \\
= & (-1)^{(i_1 -1)+[(i_2 -1)-1]+\cdots +[(i_k -1)-(k-1)]} \\
& \cdot (-1)^{\tau (\mu _1 \cdots \mu _k ) +\tau (\nu _1 \cdots \nu _{n-k}) +(j_1 -1)+[(j_2 -1)-1] +\cdots +[(j_k -1)-(k-1)]} \\
= & (-1)^{(i_1 +i_2 +\cdots +i_k )+(j_1 +j_2 +\cdots +j_k )-2(1+2+\cdots +k )+\tau (\mu_ 1 \cdots \mu _k )+\tau (\nu _1 \cdots \nu _{n-k} )} \\
= & (-1)^{\tau (\mu _1 \cdots \mu _k )} \cdot (-1)^{(i_1 +\cdots +i_k )+(j_1 +\cdots +j_k )} \cdot (-1)^{\tau (\nu _1 \cdots \nu _{n-k})}
\end{align}$$

因此$\eqref{17}$是$\det A$的一项.

:设排列$\mu _1 \cdots \mu _k$经过$s$次对换变成$j_1 \cdots j_k$.由于$j_1 \cdots j_k$是偶排列,所以$s$与$\mu _1 \cdots \mu _k$有相同奇偶性.由于在上述$s$次对换下,$\mu _1 \cdots \mu _k \nu _1 \cdots \nu _{n-k}$变成$j_1 \cdots j_k \nu _1 \cdots \nu _{n-k}$,所以

$$\begin{align}
& (-1)^{\tau (\mu _1 \cdots \mu _k \nu _1 \cdots \nu _{n-k} )} \\
= & (-1)^s \cdot (-1)^{\tau (j_1 \cdots j_k \nu _1 \cdots \nu _{n-k} )} \\
= & (-1)^{\tau (\mu _1 \cdots \mu _k )} \cdot (-1)^{(j_1 -1) +[(j_2 -1)-1]+\cdots +[(j_k -1)-(k-1)]+\tau (\nu _1 \cdots \nu _{n-k} )} .
\end{align}$$

$9$.设$A\in M_n (\mathbb{R} ),B\in M_m (\mathbb{R} )$是非退化矩阵,$C$是任意$n\times m$矩阵.利用矩阵的分块乘法(见第$2$章$\S 3$习题$17$)证明

$$\begin{pmatrix} A & C \\ 0 & B \end{pmatrix} ^{-1} =\begin{pmatrix} A^{-1} & -A^{-1}CB^{-1} \\ 0 & B^{-1} \end{pmatrix} .$$

证明:只需要把右上角变成零矩阵(形象地说,在右上角打出一个洞).办法是把矩阵的第$2$块行的$-CB^{-1}$倍加到第$1$块行上,则右上角变成$0$.这相当于在矩阵的左边乘上一个相应的分块初等矩阵,因此我们有

$$\begin{pmatrix} E_n & -CB^{-1} \\ 0 & E_m \end{pmatrix} \begin{pmatrix} A & C \\ 0 & B \end{pmatrix} =\begin{pmatrix} A & 0 \\ 0 & B \end{pmatrix} ,$$

由上式得

$$\begin{pmatrix} A & C \\ 0 & B \end{pmatrix} ^{-1} =\begin{pmatrix} A & 0 \\ 0 & B \end{pmatrix} ^{-1} \begin{pmatrix} E_n & -CB^{-1} \\ 0 & E_m \end{pmatrix} =\begin{pmatrix} A^{-1} & -A^{-1}CB^{-1} \\ 0 & B^{-1} \end{pmatrix} .$$

$10$.证明:若$A,B,C,D\in M_n (\mathbb{R} ),\det A\neq 0$则

$$\begin{align}
\det \begin{pmatrix} A & B \\ C & D \end{pmatrix} & =\det (AD-ACA^{-1}B) \\
& =(\det A)\cdot \det (D-CA^{-1}B).
\end{align}$$

此外,验证

$$\det \begin{pmatrix} A & B \\ C & D \end{pmatrix} =\begin{cases} \det (AD-CB), & 若AC=CA, \\ \det (DA-CB), & 若AB=BA. \end{cases} $$

证明:$(1)\;$当$\det A\neq 0$时,可以作下述分块矩阵的初等行变换:

$$\begin{pmatrix} A & B \\ C & D \end{pmatrix} \xrightarrow[]{F_{2,1}(-CA^{-1})} \begin{pmatrix} A & B \\ 0 & D-CA^{-1}B \end{pmatrix} ,$$

从而

$$\begin{pmatrix} E & 0 \\ -CA^{-1} & E \end{pmatrix} \begin{pmatrix} A & B \\ C & D \end{pmatrix} =\begin{pmatrix} A & B \\ 0 & D-CA^{-1}B \end{pmatrix} .$$

两边取行列式,得

$$(\det E) \cdot (\det E) \cdot \det \begin{pmatrix} A & B \\ C & D \end{pmatrix} =(\det A) \cdot \det (D-CA^{-1}B).$$

于是

$$\det \begin{pmatrix} A & B \\ C & D \end{pmatrix} =\det [A(D-CA^{-1}B)] =\det (AD-ACA^{-1}B),$$

即证

$$\begin{align}
\det \begin{pmatrix} A & B \\ C & D \end{pmatrix} & =\det (AD-ACA^{-1}B) \\
& =(\det A)\cdot \det (D-CA^{-1}B).
\end{align}$$

$(2)\;$若$AC=CA$,则根据$(1)$的结论有

$$\det \begin{pmatrix} A & B \\ C & D \end{pmatrix} =\det (AD-ACA^{-1}B) =\det (AD-CB).$$

若$AB=BA$,可以作下述分块矩阵的初等列变换:

$$\begin{pmatrix} A & B \\ C & D \end{pmatrix} \xrightarrow[F_{2,1}(-A^{-1}B)]{} \begin{pmatrix} A & 0 \\ C & D-CA^{-1}B \end{pmatrix} ,$$

从而

$$\begin{pmatrix} A & B \\ C & D \end{pmatrix} \begin{pmatrix} E & -A^{-1}B \\ 0 & E \end{pmatrix} =\begin{pmatrix} A & 0 \\ C & D-CA^{-1}B \end{pmatrix} .$$

两边右乘矩阵$\begin{pmatrix} E & 0 \\ 0 & A \end{pmatrix}$,得

$$\begin{pmatrix} A & B \\ C & D \end{pmatrix} \begin{pmatrix} E & -A^{-1}B \\ 0 & E \end{pmatrix} \begin{pmatrix} E & 0 \\ 0 & A \end{pmatrix} =\begin{pmatrix} A & 0 \\ C & D-CA^{-1}B \end{pmatrix} \begin{pmatrix} E & 0 \\ 0 & A \end{pmatrix} ,$$

$$\begin{pmatrix} A & B \\ C & D \end{pmatrix} \begin{pmatrix} E & -A^{-1}B \\ 0 & E \end{pmatrix} \begin{pmatrix} E & 0 \\ 0 & A \end{pmatrix} =\begin{pmatrix} A & 0 \\ C & DA-CA^{-1}BA \end{pmatrix} =\begin{pmatrix} A & 0 \\ C & DA-CB \end{pmatrix} ,$$

两边取行列式,得

$$\det \begin{pmatrix} A & B \\ C & D \end{pmatrix} \cdot (\det E) \cdot (\det E) \cdot (\det E) \cdot (\det A) =(\det A) \cdot \det (DA-CB).$$

因为$\det A\neq 0$,故

$$\det \begin{pmatrix} A & B \\ C & D \end{pmatrix} =\det (DA-CB).$$

即证

$$\det \begin{pmatrix} A & B \\ C & D \end{pmatrix} =\begin{cases} \det (AD-CB), & 若AC=CA, \\ \det (DA-CB), & 若AB=BA. \end{cases} $$

文章目錄
  1. 1. 非退化矩阵的判别准则
  2. 2. 克拉默公式
  3. 3. 加边子式法
  4. 4. 习题